Question

4. For this question you will be analyzing a market where firms compete under Bertrand com petition. That is, firms will strategically compete by selecting prices in order to maximize their profit. For this market, let the market demand be o 50-2p (a) Suppose firm I has a marginal cost of 10 and firm 2 has a marginal cost of 5. What is the equilibrium price p., what are the equilibrium quantities the firms produce q1-q2, and what is the equili brium total quantity? 3 points) (b) Now consider a setup where firm 1 and firm 2 have the same marginal costs, but now there is also a firm 3 with a marginal cost of 4. What is the equilibrium price p, what are the equilibium quantities the firms produce qi 2 and what is the equilibrium total quantity? (4 points) (c) Now consider a setup where firm 1, firm 2, and firm 3 have the same marginal costs but now there is also a firm 4 with a marginal cost of 4. What is the equili brium price p, what are the equilibnuin quantities the firms produce qi, q ,%-and what is the equilibrium total quantity? (5 points) (d) What if there was a capacity constraint that limited all firms production to at most 10 units? Revisit the setup from part a (only firm 1 and firm 2) and find the equilibrium price, firm quantities, and total quantity with the capacity constraint. (4 points)

0 0
Add a comment Improve this question Transcribed image text
Request Professional Answer

Request Answer!

We need at least 10 more requests to produce the answer.

0 / 10 have requested this problem solution

The more requests, the faster the answer.

Request! (Login Required)


All students who have requested the answer will be notified once they are available.
Know the answer?
Add Answer to:
4. For this question you will be analyzing a market where firms compete under Bertrand com...
Your Answer:

Post as a guest

Your Name:

What's your source?

Earn Coins

Coins can be redeemed for fabulous gifts.

Similar Homework Help Questions
  • Consider a market where N firms produce a homogeneous product and compete by simultaneously setting quantities....

    Consider a market where N firms produce a homogeneous product and compete by simultaneously setting quantities. The inverse demand function has the general form P PO-P(qi +q2 +q3 + + qv), where Q is total quantity produced, qi is the quantity produced by firm i and P is the market price. The demand curve is downward sloping, so P10 < 0. The total cost of firm i is given by Cig). (0) Show that P- MC qi i , where...

  • 3. There are two firms that compete according to Cournot competition. Firm 1 has a cost...

    3. There are two firms that compete according to Cournot competition. Firm 1 has a cost function G(91) = 5.59+12. Firm 2 has a cost function C(q2) = 2.5q3 + 18. These firms cannot discriminate, so there is just one price that is determined by the aggregate demand. The inverse demand equation is P(Q) = 600 – 0 Where total supply Q-q1+92. (e) Use your best response equations to mathematically solve for the equilibrium quantities qi 9, Q". equilibrium price...

  • Two firms sell identical products and compete as Cournot (price-setting) competitors in a market with a...

    Two firms sell identical products and compete as Cournot (price-setting) competitors in a market with a demand of p = 150 - Q. Each firm has a constant marginal and average cost of $3 per unit of output. Find the quantity each firm will produce and the price in equilibrium.

  • 4. Bertrand Competition (29 points) Consider a Betrand Model. The market demand is P=130-Q. Consumers only...

    4. Bertrand Competition (29 points) Consider a Betrand Model. The market demand is P=130-Q. Consumers only buy from the firm charging a lower price. If the two firms charge the same price. they share the market equally. The marginal cost for firm 1 is 10, and the marginal cost for firm 2 is also 10. There are no fixed costs. A. (5 points) Would any firm charge a price below 10 at the market equilibrium? Briefly explain your reason. B....

  • Exercise 5 Let us consider a market where 4 firms compete à la Bertrand. The demand...

    Exercise 5 Let us consider a market where 4 firms compete à la Bertrand. The demand function is given by q() = 250 - 7p. The cost function is the same for both firms and it is C(q) = 100; for all i E {1,2,3,4} • Write explicitly the demand and profit functions of 1, 2, 3, and 4. • Derive best reply functions and the Nash equilibrium of the game. (9) = 591, what • If firm 1 find...

  • Question 2: Simultaneous quantity choiceTwo firms F1 and F2 produce a homogeneous product and compete on...

    Question 2: Simultaneous quantity choiceTwo firms F1 and F2 produce a homogeneous product and compete on the same market. The market price is described by the inverse demand curveP= 11−2Q, where Q is total industry output andPis the market price. To keep things simple, suppose that each firm can produce either 1 or 2 units (these are the only possible choices of production).Further suppose that both firms have a constant marginal cost equal to 2, so that the total cost...

  • Problem 1: Suppose that the market demand function is given by q-80-2p. All firms in the...

    Problem 1: Suppose that the market demand function is given by q-80-2p. All firms in the industry have marginal cost of 10 and no fixed cost. In this problem, the firms compete in quantities. (a) What is the equilibrium price, quantity, consumer surplus, profit (producer surplus) and deadweight loss if there is only one firm in the industry? (b) Now answer the same question if there are two firms in the industry (duopoly). How does your answer compare to the...

  • 4. Bertrand Competition (29 points) Consider a Betrand Model. The market demand is P-130-Q, Consumers only buy from...

    4. Bertrand Competition (29 points) Consider a Betrand Model. The market demand is P-130-Q, Consumers only buy from the firm charging a lower price. If the two firms charge the same price, they share the market equally. The marginal cost for firm 1 is 10, and the marginal cost for firm 2 is also 10. There are no fixed costs. A. (5 points) Would any firm charge a price below your reason. at the market equilibrium? Briefly explain B. (6...

  • Two profit-maximizing firms compete in a market. Firm 1 chooses quantity qı > 0 and Firm...

    Two profit-maximizing firms compete in a market. Firm 1 chooses quantity qı > 0 and Firm 2 chooses quantity 42 > 0. The market price is: p(91,92) = 8 - 2q1 - 42. The cost to Firm 1 of producing qi is C1 = 41. The cost to Firm 2 of producing 92 is C2 = 42 + 42. a.) * Calculate the best-response function for each firm. b.) Suppose the two firms choose their quantities simultaneously. What is the...

  • MomCorp (M) and Planet Express, Inc. (E) are two firms that compete in a Cournot duopoly...

    MomCorp (M) and Planet Express, Inc. (E) are two firms that compete in a Cournot duopoly by simultaneously setting quantities (of package deliveries). Denote MomCorp's quantity by Q and Planet Express's quantity by Qe. The package deliveries they offer are identical, so the price is determined in a combined market according to the inverse demand equation, P = 120-Q, where Q = Qu + Qe. Suppose that MomCorp has constant marginal cost, MCM = 20, while Planet Express has constant...

ADVERTISEMENT
Free Homework Help App
Download From Google Play
Scan Your Homework
to Get Instant Free Answers
Need Online Homework Help?
Ask a Question
Get Answers For Free
Most questions answered within 3 hours.
ADVERTISEMENT
ADVERTISEMENT
ADVERTISEMENT